Mathcenter Forum  

Go Back   Mathcenter Forum > คณิตศาสตร์โอลิมปิก และอุดมศึกษา > พีชคณิต
สมัครสมาชิก คู่มือการใช้ รายชื่อสมาชิก ปฏิทิน ข้อความวันนี้

ตั้งหัวข้อใหม่ Reply
 
เครื่องมือของหัวข้อ ค้นหาในหัวข้อนี้
  #1  
Old 19 มิถุนายน 2005, 12:30
M@gpie's Avatar
M@gpie M@gpie ไม่อยู่ในระบบ
ลมปราณไร้สภาพ
 
วันที่สมัครสมาชิก: 09 ตุลาคม 2003
ข้อความ: 1,227
M@gpie is on a distinguished road
Post พหุนามคร้าบ

เอามาจากข้อสอบสอวน.
1. ให้ \( \ \ a,b,c,d \ \ \) เป็นรากทั้งสี่ของสมการ \( x^4 -3x+1 =0 \)
และ ถ้า \( f(x) = x^4 +a_1 x^3 +a_2 x^2 +a_3x +a_4\) เป็นพหุนามที่มี
\( \frac{a+b+c}{d^2} , \frac{a+b+d}{c^2} , \frac{a+c+d}{b^2} , \frac{b+c+d}{a^2}\) เป็นรากของสมการ \( f(x) = 0 \)
แล้ว จงหาว่า \( 7+a_1+a_2+a_3+a_4 \) มีค่าเท่าใด

2.ถ้า x เป็นจำนวนจริงบวก แล้ว จงหาค่าต่ำสุดของ \[f(x) = \frac{\left( x+\frac{1}{x} \right)^6 - \left( x^6 + \frac{1}{x^6} \right) - 2 }{(x +\frac{1}{x})^3 + x^3 +\frac{1}{x^3} } \]

3.ให้ \( g(x) = x^5 +x^4+x^3+x^2+x+1 \)
และ \( r(x) , q(x) \) เป็นพหุนามซึ่ง \( g(x^{12}) = g(x)q(x) + r(x)\) โดยที่ดีกรีของ \( r(x) \) น้อยกว่า 5 แล้วจงหาค่าของ
\( r(1) + r(2) + r(3) + r(4) \)
__________________
PaTa PatA pAtA Pon!

20 มิถุนายน 2005 19:14 : ข้อความนี้ถูกแก้ไขแล้ว 3 ครั้ง, ครั้งล่าสุดโดยคุณ M@gpie
ตอบพร้อมอ้างอิงข้อความนี้
  #2  
Old 19 มิถุนายน 2005, 23:48
R-Tummykung de Lamar R-Tummykung de Lamar ไม่อยู่ในระบบ
กระบี่ประสานใจ
 
วันที่สมัครสมาชิก: 20 ธันวาคม 2004
ข้อความ: 566
R-Tummykung de Lamar is on a distinguished road
Post

ขอประเดิมข้อ 1 เลยละกันนะครับ
ให้ \( \displaystyle{x^4-3x+1\ =\ 0}\) เป็นสมการ (1)
จาก a,b,c และ d เป็นรากของสมการนี้จะได้ a+b+c+d = 0
\(\therefore\ \displaystyle{\frac{a+b+c}{d^2}\ =\ \frac{-d}{d^2}\ =\ -\frac{1}{d}}\ \ \ \) อันอื่นก็ทำนองเดียวกันได้ รากของสมการ f(x)=0 คือ \( \displaystyle{-\frac{1}{a}} \) ,\( \displaystyle{-\frac{1}{b}} \) ,\( \displaystyle{-\frac{1}{c}} \) และ \( \displaystyle{-\frac{1}{d}} \)
\( \displaystyle{\begin{array}{rcll} ดังนั้น f(x)&=&\big(x+\frac{1}{a}\big)\big(x+\frac{1}{b}\big)\big(x+\frac{1}{c}\big)\big(x+\frac{1}{d}\big)\\&=&x^4+\big(\frac{1}{a}+\fr ac{1}{b}+\frac{1}{c}+\frac{1}{d}\big)x^3+\big(\frac{ 1}{ab}+\frac{1}{ac}+\frac{1}{ad}+\frac{1}{bc}+\frac{1}{bd}+\frac{1}{cd}\big)x^2\\&&+\big(\frac{1}{abc}+\frac{1}{abd}+\frac{1}{bc d}\big)x+\big(\frac{1}{abcd}\big)\\a_1+a_2+a_3+a_4&= &\frac{1+(a+b+c+d)+(ab+ac+ad+bc+bd+cd)+(abc+abd+bcd)+(abcd)}{abcd}\\ &=&\frac{1-(0)+(0)-(-3)+(1)}{1}\\&=&5 \end{array}} \)
ดังนั้น 7+a1+a2+a3+a4 = 12

แต่ผมดูตัวเลขแล้ว น่าจะมีวิธีที่ดีกว่านี้นะครับ
__________________
[[:://R-Tummykung de Lamar\\::]] ||
(a,b,c > 0,a+b+c=3)
$$\sqrt a+\sqrt b+\sqrt c\geq ab+ac+bc$$
ตอบพร้อมอ้างอิงข้อความนี้
  #3  
Old 20 มิถุนายน 2005, 01:41
passer-by passer-by ไม่อยู่ในระบบ
ผู้พิทักษ์กฎทั่วไป
 
วันที่สมัครสมาชิก: 11 เมษายน 2005
ข้อความ: 1,442
passer-by is on a distinguished road
Post

อ้างอิง:
ข้อความเดิมของ R-Tummykung De Lamar:

\(\large a_{1} + a_{2}+a_{3} + a_{4}=\frac{1+(a+b+c+d)+(ab+ac+ad+bc+bd+cd)+(abc+abd+bcd)+abcd}{abcd} \)
ที่ตัวเศษ ไม่มี abcd นะครับ น้อง tummy

ส่วนวิธีสั้นๆ อาจทำได้ดังนี้ครับ หลังจากรู้แล้วว่า -1/a,...-1/d เป็นราก f(x)=0
ถ้าให้ y= -1/x เมื่อ x เป็นรากของ x4-3x+1=0
จะสามารถสร้าง f(x) ได้เป็น\[\large \bigg (\frac{-1}{y}\bigg)^{4}-3\bigg(\frac{-1}{y}\bigg)+1=0 \] หรือ simplify เป็น y4+3y3+1=0
แสดงว่า a1=3 , a2= a3=0, a4=1

ดังนั้น คำตอบที่ต้องการคือ 11
__________________
เกษียณตัวเอง ปลายมิถุนายน 2557 แต่จะกลับมาเป็นครั้งคราว
ตอบพร้อมอ้างอิงข้อความนี้
  #4  
Old 20 มิถุนายน 2005, 19:13
M@gpie's Avatar
M@gpie M@gpie ไม่อยู่ในระบบ
ลมปราณไร้สภาพ
 
วันที่สมัครสมาชิก: 09 ตุลาคม 2003
ข้อความ: 1,227
M@gpie is on a distinguished road
Post

ผมใช้วิธีเดียวกับคุณ passer-by นะคร้าบบ คำตอบคือ 11 ดังแสดง
แต่อีกสองข้อ สารภาพว่ายังคิดไม่ออกเลยครับ
__________________
PaTa PatA pAtA Pon!
ตอบพร้อมอ้างอิงข้อความนี้
  #5  
Old 20 มิถุนายน 2005, 19:55
gon's Avatar
gon gon ไม่อยู่ในระบบ
ผู้พิทักษ์กฎขั้นสูง
 
วันที่สมัครสมาชิก: 29 มีนาคม 2001
ข้อความ: 4,608
gon is on a distinguished road
Smile

ข้อ 2 ก็ไม่ยากครับ. สมมติให้ \((x + \frac{1}{x})^3 = a, x^3 + \frac{1}{x^3} = b \)

ดังนั้น \(f(x) = \frac{a^2 - (b^2 - 2) - 2}{a + b} = a - b = (x + \frac{1}{x})^3 - (x^3 + \frac{1}{x^3}) = 3(x + \frac{1}{x}) \geq 3(2) = 6 \)
ตอบพร้อมอ้างอิงข้อความนี้
  #6  
Old 20 มิถุนายน 2005, 22:20
passer-by passer-by ไม่อยู่ในระบบ
ผู้พิทักษ์กฎทั่วไป
 
วันที่สมัครสมาชิก: 11 เมษายน 2005
ข้อความ: 1,442
passer-by is on a distinguished road
Post

ข้อ 3 ผมว่า r(x) น่าจะเป็น constant function เพราะ
g(x) หาร x6-1 ลงตัว
และขณะเดียวกัน \[\large \begin{array}{lc}
g(x^{12})= x^{60}+ x^{48}+x^{36}+ x^{24}+ x^{12}+1\\ \qquad=(x^{60}-1)+ (x^{48}-1)+(x^{36}-1)+(x^{24}-1)+ (x^{12}-1)+6\\\qquad= A+B+C+D+E+6 \end{array}\]
แต่ x6-1 หาร A,B,C,D,E ลงตัว
ดังนั้น g(x) หาร A,B,C,D,E ลงตัว ด้วย ซึ่งเมื่อรวมผลหารเข้าด้วยกันก็จะเป็น q(x)
ที่เหลือก็คือเศษ ซึ่งก็คือ r(x) = 6
ข้อนี้จึงน่าจะตอบ 24 (ไม่รู้ถูกหรือเปล่า)
__________________
เกษียณตัวเอง ปลายมิถุนายน 2557 แต่จะกลับมาเป็นครั้งคราว
ตอบพร้อมอ้างอิงข้อความนี้
ตั้งหัวข้อใหม่ Reply



กฎการส่งข้อความ
คุณ ไม่สามารถ ตั้งหัวข้อใหม่ได้
คุณ ไม่สามารถ ตอบหัวข้อได้
คุณ ไม่สามารถ แนบไฟล์และเอกสารได้
คุณ ไม่สามารถ แก้ไขข้อความของคุณเองได้

vB code is On
Smilies are On
[IMG] code is On
HTML code is Off
ทางลัดสู่ห้อง


เวลาที่แสดงทั้งหมด เป็นเวลาที่ประเทศไทย (GMT +7) ขณะนี้เป็นเวลา 15:46


Powered by vBulletin® Copyright ©2000 - 2024, Jelsoft Enterprises Ltd.
Modified by Jetsada Karnpracha